You are on page 1of 21
Solutions Manual to Walter Rudin’s Principles of Mathematical Analysis Roger Cooke, University of Vermont Chapter 3 Numerical Sequences and Series Exercise 3.1 Prove that convergence of {s,,} implies convergence of {|p|}. Is the converse true? Solution. Let ¢ > 0. Since the sequence {s,} is a Cauchy sequence, there exists N such that |Sm—q| <¢ for all m > N and n > N. We then have |I5ml — [nll < [Sm — Sn] < for all m > N and n > N. Hence the sequence {|sn|} is also a Cauchy sequence, and therefore must converge. The converse is not true, as shown by the sequence {sq} with 5, Exercise 3.2 Calculate lim (Vn? +n ~n) Solution. Multiplying and dividing by Vn? =n +n yields Vrbtn n Vt tntn It follows that the limit is }. Exercise 3.3 If s; = V2 and Peace cereee ree dees eee prove that {s,} converges, and that s, <2forn=1,2,3.... Solution. Since V2 < 2, it is manifest that if s, < 2, then say, < V2+2=2. Hence it follows by induction that V2 < s, < 2 for all n. In view of this fact, 29 30 CHAPTER 3. NUMERICAL SEQUENCES AND SERIES it also follows that (sp — 2)(8n +1) <0 for alm > 1, ie, 8p > 923-2 = Spa Hence the sequence is an increasing sequence that is bounded above (by 2) and so converges. Since the limit s satisfies s? — s ~ 2 = 0, it follows that the limit is 2 Exercise 3.4 Find the upper and lower limits of the sequence {sn} defined by 81 =0; 83m = Solution. We shall prove by induction that il 1 Som = 5 ~ Gye aD Sams = 1 Fe for m = 1,2,.... The second of these equalities is a direct consequence of the firs $2 , and so we need only prove the first. Immediate computation shows that and s3 = 4. Hence assume that both formulas hold for m

You might also like